Jump to content

poles of an electromagnet question


DandelionTheory

Recommended Posts

39 minutes ago, DandelionTheory said:

I didn't put it here. It was moved. I made statements with math and it was still moved to speculation. 

I don't know when exactly it was moved (or by whom) but may well have been when you said:

On 1/31/2020 at 11:10 AM, DandelionTheory said:

So you just assume they [Newton's Laws] apply to everything you think, and there is no way around any of it even with addition or cancelation.

 

40 minutes ago, DandelionTheory said:

For an op to ask questions about physics in a classical physics forum is not off base, but to assume any point being made is base for the whole post to be sent to speculation... Best move. I don't know what the forum is for if not to ask questions, and for any point to be assumed speculation isn't a forum it's gatekeeping.

But you are not just asking questions, you are also making assertions contradicted by physics.

Link to comment
Share on other sites

22 minutes ago, Strange said:

But you are not just asking questions, you are also making assertions contradicted by physics

I see you think it's contradictory. I was making a point on his character not what he was saying. I never asserted anything other than a force on a current in a direction. I didn't make assertions I had a solution. I did not claim this idea violated any law, nor does it violate any known ones. Because I mentioned a belief of another person I was moved? Lame.

Edited by DandelionTheory
Link to comment
Share on other sites

2 hours ago, DandelionTheory said:

Because I mentioned a belief of another person I was moved?

1. Accepting that Newton's laws apply universally is not a "belief". It is mainstream science, based on evidence.

2. I don't know if that is why the thread was moved.

Link to comment
Share on other sites

Dandelion theory have you made the connection to Gideon's question concerning dot product of two vectors and the cross product of two vectors of the Lorentz force in terms of the right hand rule ?

It is essential you understand this in regards to the formula for the Lorentz force.

If you have a wire perpendicular to a current carrying wire then the force on the wire will be a cross product. So the equation you will need will also require an angle between the two. 

PS this applies to both your threads 

Edited by Mordred
Link to comment
Share on other sites

 

On 2/1/2020 at 2:55 PM, DandelionTheory said:

Please allow me to aggregate the needed variables.

Ok!

 

A note regering the assumption:

On 12/22/2019 at 12:26 AM, DandelionTheory said:

I'm assuming this produces a momentum change in the positive Y direction, but i can be wrong.

To reduce the risk that there is a misunderstanding, exactly what is assumed ho have change of momentum? Is the blue circular cable free to slide upwards out of the gap or is the blue cable mounted solid and supposed to be staying in the gap?

 

Link to comment
Share on other sites

On 2/1/2020 at 9:28 AM, Mordred said:

Dandelion theory have you made the connection to Gideon's question concerning dot product of two vectors and the cross product of two vectors of the Lorentz force in terms of the right hand rule ?

It is essential you understand this in regards to the formula for the Lorentz force.

If you have a wire perpendicular to a current carrying wire then the force on the wire will be a cross product. So the equation you will need will also require an angle between the two. 

PS this applies to both your threads 

Yes I know, observe the uniformity of the B fields and apply. The lorentz force has a sin theta variable, while the rail gun calculation involves 3 conductors with uniform fields some variables cancel. It also assumes you understand the right hand rule.

 

On 2/2/2020 at 8:52 AM, Ghideon said:

Is the blue circular cable free to slide upwards out of the gap or is the blue cable mounted solid and supposed to be staying in the gap?

It slides. Does it matter though? because the reaction on the core would be opposite ( net force down). I guess seeing as the blue cable can move and since it's obviously less dense than the core, it would move up. (Assuming)

It is exactly opposite forces in direction and magnitude.

Edited by DandelionTheory
Link to comment
Share on other sites

Let's use a generic description. The cross product of a vector is perdendicular while the dot product is parallel

The magnetic field has a curl so what are the ramifications to the above in regards to a spinor ?

Edited by Mordred
Link to comment
Share on other sites

3 hours ago, DandelionTheory said:

It slides. Does it matter though? because the reaction on the core would be opposite ( net force down).

I believe it matters since calculations will be different, the force is not constant. The blue cable will slide upwards, and after a short time slide out of the gap and experience a different magnetic field. You would have to calculate force as a function of time. That may be tricky (haven't tried yet). 

And it is correct, the core will be pushed down when the cable is pushed up. 

3 hours ago, DandelionTheory said:

I guess seeing as the blue cable can move and since it's obviously less dense than the core, it would move up. (Assuming)

 If there is a current in the blue wire I assume it is powered somehow and connected to something, affecting movements. But in an ideal situation we can choose to neglect that to simplify. 

 

On 12/22/2019 at 12:26 AM, DandelionTheory said:

I'm assuming this produces a momentum change in the positive Y direction

Again, what will have a momentum? I guess you mean the blue cable? If so then yes (at least for a short while).


The total momentum of cable + the core is conserved, the combined center of mass for cable and core will not move.

 

Link to comment
Share on other sites

1 hour ago, Ghideon said:

The total momentum of cable + the core is conserved, the combined center of mass for cable and core will not move

Thank you. You pointed out my claim better than I did. I have another inquiry;

Does the initial angle of interaction between the current and the core's magnetic field affect the magnitude of the interaction due to it being 3 bodies of interaction? Ie if the angle between the poles was 10° but the B vector adds to 0°, does it change the magnitude of the interaction?

Edited by DandelionTheory
Affect
Link to comment
Share on other sites

32 minutes ago, DandelionTheory said:

Does the initial angle of interaction between the current and the core's magnetic field affect the magnitude of the interaction due to it being 3 bodies of interaction? Ie if the angle between the poles was 10° but the B vector adds to 0°, does it change the magnitude of the interaction?

Changing an angle between current and the magnetic fields may change the outcome when calculating the force. But the total momentum of cable + the core is conserved, the combined center of mass for cable and core will not move

Maybe you could provide a picture? 

Link to comment
Share on other sites

1 hour ago, DandelionTheory said:

Is there a different interaction between straight magnetic fields and round ones than 2 round ones?

Hard to give a generally applicable answer, depends on directions and magnitudes. Calculations may be required.   In a simple and static case I would add the vectors of the involved magnetic fields and then calculate the effects from the resulting field. 

Link to comment
Share on other sites

11 hours ago, Ghideon said:

Hard to give a generally applicable answer, depends on directions and magnitudes. Calculations may be required.   In a simple and static case I would add the vectors of the involved magnetic fields and then calculate the effects from the resulting field. 

Thank you. I'm having trouble finding sources to address this specifically. A problem I run into is most people that explain magnetic field interaction attempt to translate them into moving currents. While it's a novel idea and proved to work, I think it misses the point with the difference between them being round and or straight. A loop of current has a pivot point in the center and not centered at the current itself, I'm attempting to see if it makes a difference.

Edited by DandelionTheory
Link to comment
Share on other sites

1 hour ago, DandelionTheory said:

Thank you. I'm having trouble finding sources to address this specifically. A problem I run into is most people that explain magnetic field interaction attempt to translate them into moving currents. While it's a novel idea and proved to work, I think it misses the point with the difference between them being round and or straight.

It's been a (long) while since I studied electromagnetic field theory so my sources are of limited use. Depending on background additional studies of math may be required, for instance to calculate the effect of superimposed fields or charges distributed across surfaces or volumes. Basically it is a matter of describing the shape of the field(s) and apply the correct math. Depending on the shape and/or time dependencies the math may be very simple or really hard. The physics is well understood and documented in lots of books, typically with examples of linear or curved magnetic fields. 

1 hour ago, DandelionTheory said:

A loop of current has a pivot point in the center and not centered at the current itself, I'm attempting to see if it makes a difference.

A difference compared to what?

Link to comment
Share on other sites

1 hour ago, Ghideon said:

A difference compared to what?

Compared to one that doesn't.

Loops of current will pivot about their axis to align with another magnetic field, which I assume requires energy.

Protons rotate about their center axis towards the electric field, while a loop of current rotates about their center axis towards the opposite signed magnetic field.

I want to know if equal but opposite forces can be transformed between the electric and magnetic field.

Link to comment
Share on other sites

3 hours ago, DandelionTheory said:

Compared to one that doesn't.

Loops of current will pivot about their axis to align with another magnetic field, which I assume requires energy.

Protons rotate about their center axis towards the electric field, while a loop of current rotates about their center axis towards the opposite signed magnetic field.

I would have to see a picture, hard to visualize what you are proposing/asking.

 

3 hours ago, DandelionTheory said:

I want to know if equal but opposite forces can be transformed between the electric and magnetic field.

 This is the first time in this thread electric fields are mentioned in this thread, I'm not able to find enough information to comment. 

Link to comment
Share on other sites

 

5 hours ago, Ghideon said:

I would have to see a picture, hard to visualize what you are proposing/asking.

I misspoke. This illustrates the idea with pivot points and a Coulomb's law calculation on 5 bodies. Assume line FG is rigid.

716575259_20200206_1149102.thumb.jpg.38af4828d547e1a3ddfb7dae1a124f9d.jpg

1511562518_20200206_1054362.thumb.jpg.085f319f1f338b0274af103c91785bb5.jpg

Edited by DandelionTheory
Link to comment
Share on other sites

1 hour ago, DandelionTheory said:

This illustrates the idea with pivot points and a Coulomb's law calculation on 5 bodies

Ok.  

image.png.f37651cd349c5a0cbc01e612a86462c8.png

The formulas seem a little off. The charged circles will move relative to one another if the beams are free to swing around. That means the distance between the charges will change. Since charges move there will not be one single "r" and the values of the different "r" will be time dependent. After some time I think the system will settle with charges D,A,E,B,C aligned horizontally. 

 

How is this related to the other stuff in the thread?

 

Edited by Ghideon
fixed the order of the charges
Link to comment
Share on other sites

48 minutes ago, Ghideon said:

How is this related to the other stuff in the thread?

Pushing on a hinge.

If at the time circle A and circle C reach line FG someone were to manually change circle E to the opposite charge, the momentum of A, B, C and D will carry over to the other side of the cycle.

55 minutes ago, Ghideon said:

After some time I think the system will settle with charges D,A,E,B,C aligned horizontally. 

At this time, flip E's charge to continue.

Link to comment
Share on other sites

31 minutes ago, DandelionTheory said:

Pushing on a hinge.

?

31 minutes ago, DandelionTheory said:

If at the time circle A and circle C reach line FG someone were to manually change circle E to the opposite charge, the momentum of A, B, C and D will carry over to the other side of the cycle.

You did not say anything about switching charges. Switching the charges makes the calculations you provided less correct. Maybe you could explain what you wish to achieve? Maybe a complete drawing, all time dependencies stated and math that matches that?

33 minutes ago, DandelionTheory said:

At this time, flip E's charge to continue.

Charges can be flipped and movement made to continue. Are you trying to show a basic electric motor? 

Link to comment
Share on other sites

11 hours ago, Ghideon said:

How is this related to the other stuff in the thread

The thread states the lorentz force pushes the op current up, later you point out the opposite force on the magnet core.

I point out after there might be a significant difference in round vs straight magnetic fields. You wanted a diagram, I did a diagram with coulombs law and mentioned rotation which would imply a time variant.

I can do it with current too buddy. 

Circle E is forced up half the cycle, while no force is experienced by circle E while circle A & C are parallel to line FG.

As circles A & C pass line FG, if circle E were to be manually switched to it's opposite charge it would once again experience force for the remainder of the cycle while circles A, C, B & D rotate.

Man you know I can't calculate over time yet.

I'm still in pre calc

Edited by DandelionTheory
Link to comment
Share on other sites

2 hours ago, DandelionTheory said:

The thread states the lorentz force pushes the op current up, later you point out the opposite force on the magnet core.

I point out after there might be a significant difference in round vs straight magnetic fields. You wanted a diagram, I did a diagram with coulombs law and mentioned rotation which would imply a time variant.

Thanks for clarifying. 

2 hours ago, DandelionTheory said:

Circle E is forced up half the cycle, while no force is experienced by circle E while circle A & C are parallel to line FG.

As circles A & C pass line FG, if circle E were to be manually switched to it's opposite charge it would once again experience force for the remainder of the cycle while circles A, C, B & D rotate.

Yes there will be force(s) on circle E and they can be calculated. And by changing the charges with correct timing you could generate rotation about points F and G. I believe this is well known facts in physics. And when calculating the sum of forces in the device, not E in isolation, the forces will cancel. 

Just to state the obvious, the device will not move anywhere. It may wobble or jump due to imbalance but the common center of mass will not move. 

 

Edited by Ghideon
deleted an incorrect word
Link to comment
Share on other sites

11 hours ago, Ghideon said:

And when calculating the sum of forces in the device, not E in isolation, the forces will cancel. 

Can you show me how I would calculate that balance?

Quote

Just to state the obvious, the device will not move anywhere. It may wobble or jump due to imbalance but the common center of mass will not move

I'm having trouble finding the right documentation for linear motion on a wheel axle due to different angular torques applied. If you know of anything I would appreciate it. Because I want to know why you came to the conclusion.

 

I just realized I did the statement wrong. Apologies. I'm making a correction to this statement:

13 hours ago, DandelionTheory said:

Circle E is forced up half the cycle, while no force is experienced by circle E while circle A & C are parallel to line FG.

As circles A & C pass line FG, if circle E were to be manually switched to it's opposite charge it would once again experience force for the remainder of the cycle while circles A, C, B & D rotate

Should say:

Circle E is forced up half the cycle, while no force is experienced by circle E while circle A & B are parallel to line FG.

As circles A & B pass line FG to the  closest point of rotation to circle E, if circle E were to be manually switched to it's opposite charge it would once again experience force for the remainder of the cycle while circles A, C, B & D rotate

Edited by DandelionTheory
Link to comment
Share on other sites

1 hour ago, DandelionTheory said:

Can you show me how I would calculate that balance?

I could show, but a problem is that it requires math that you have said that you do not yet master. If I would take the time to write down the lagrangian of the specified system, resulting in differential equations predicting the motions, would you benefit from it? (Lagrangian mechanics is fun and interesting but not the topic of this thread) Instead I’ll try simplified explanation.
Some notes: We do not have to do the calculations to predict the outcome on the system as a whole.
Note that this explanation is extremely general. No matter what kind of contraption you come up with the following explanation should apply.
Note that It applies to all forces, it is not limited to electromagnetism. 

5sETJg-kB_qayX6EVKNkQL-V3TX_FLN1j7G6bhnq1-wGoe99EHqBffEbLTo96dXy2oGqv5EmAfc_6Z-cHuohgd7b7VhEKOk0R4h9FI0G2h1L6KNTk4SgkXzXmiQPEMolVUKwuQdk

Above is your system. I’ve drawn it in a blue box to highlight the boundaries.
1: According to the drawing there are no external forces pushing on the system.
2: The system is supposed to stay intact, you have not stated that it will break apart.
3: Pieces may move relative to one another but no piece will ever leave the system ad get ejected outside the rectangle.
The above constraints means that we can apply F=ma to the complete set of things in the box. And since F=0 then a=0. There is no force and hence no acceleration.
That the system does not accelerate means that whatever internal forces that we look at have to cancel. How?
Because any force experienced by any piece or part of the system in the blue rectangle will cause an equal and opposite force on some other piece or part in the box.
If the forces do not balance out internally then we would have some piece (lets call it P1) where there is a net force. A force F>0 on P1 means that according to F=ma P1 is accelerated. But what happens to the other piece (P2) in the system, experiencing the equal and opposite force? P2 will also accelerate according* to F=ma but in the other direction. The important thing is that the acceleration of the second piece P2 directed away from the first piece P1. In other words: if the internal forces does not balance it means that the system breaks apart. That does not match constraint 2. The above length reasoning can be stated mathematically, very generally applicable. 

 

I'll try an analogy using the https://en.wikipedia.org/wiki/Eiffel_Tower. AFAIK the tower remains stationary relative to earth. It does not take of as a rocket. That means that the total sum of forces on the tower is zero. If not, the tower would experience acceleration relative to earth. Now look at the internal forces. There are thousands of struts and bolts, calculating each and every force is possible but not an easy task. But we do not have to, to be able to predict the sum. We know the sum is zero since the tower is not accelerating. If we remove a strut, tighten a bolt or add some magnets or stuff in the tower's complicated structure it will never ever make the tower fly. F=0 for the tower whatever complicated rearrangement we ever try. What we can do is removing or adding stuff or shifting things internally so that some part(s) of the tower breaks. Then various parts will fall in various directions. But never ever will we be able to shift forces internally in the towers structure so that tower moves it’s center of mass and stays intact. 

Again this is all about classical mechanics. If we do the calculations in detail of your system and find some net force we know that the calculation is incorrect. 

The explanation is valid for low masses and low speeds, relativistic effects is not considered.

 

1 hour ago, DandelionTheory said:

I'm having trouble finding the right documentation for linear motion on a wheel axle due to different angular torques applied. If you know of anything I would appreciate it. Because I want to know why you came to the conclusion.

I'll see what I can find.

 

*) If the masses of the pieces P1 and P2 differ the magnitude of acceleration will differ.

 


 

Link to comment
Share on other sites

Guest
This topic is now closed to further replies.
×
×
  • Create New...

Important Information

We have placed cookies on your device to help make this website better. You can adjust your cookie settings, otherwise we'll assume you're okay to continue.